You are on page 1of 53

Insights Mock tests – 2015: test 9 Solutions

1. Solution: c)

As can be clearly seen agriculture had developed and spread long back. It was a not a cause
for the rise in population. Low death rates due to better sanitation facilities; and
industrialization increased economic wealth that led to better living standards and a rise in
population.

2. Solution: b)

The rock shelters on banks of the River Suyal at Lakhudiyar, about twenty kilometres on the
Almora-Barechina road (Uttarkhand), bear these prehistoric paintings. Lakhudiyar literally
means one lakh caves. The paintings here can be divided into three categories: man, animal
and geometric patterns in white, black and red ochre. Humans are represented in stick-like
forms. A long-snouted animal, a fox and a multiple legged lizard are the main animal
motifs. Wavy lines, rectangle-filled geometric designs, and groups of dots can also be seen
here. One of the interesting scenes depicted here is of hand-linked dancing human figures.
There is some superimposition of paintings. The earliest are in black; over these are red
ochre paintings and the last group comprises white paintings.

The richest paintings are reported from the Vindhya ranges of Madhya Pradesh and their
Kaimurean extensions into Uttar Pradesh. These hill ranges are full of Palaeolithic and
Mesolithic remains, and they are also full of forests, wild plants, fruits, streams and creeks,
thus a perfect place for Stone Age people to live. Among these the largest and most
spectacular rock-shelter is located in the Vindhya hills at Bhimbetka in Madhya Pradesh.
Bhimbetka is located fortyfive kilometres south of Bhopal, in an area of ten square
kilometres, having about eight hundred rock shelters, five hundred of which bear paintings.

3. Solution: d)

An important characteristic of all communities is that composition and structure constantly


change in response to the changing environmental conditions. This change is orderly and
sequential, parallel with the changes in the physical environment. These changes lead finally

http://insightsonindia.com INSIGHTS Page 1


Insights Mock tests – 2015: test 9 Solutions

to a community that is in near equilibrium with the environment and that is called a climax
community.

The gradual and fairly predictable change in the species composition of a given area is called
ecological succession. During succession some species colonise an area and their populations
become more numerous, whereas populations of other species decline and even disappear.
The entire sequence of communities that successively change in a given area are called
sere(s).

The individual transitional communities are termed seral stages or seral communities. In the
successive seral stages there is a change in the diversity of species of organisms, increase in
the number of species and organisms as well as an increase in the total biomass. The present
day communities in the world have come to be because of succession that has occurred over
millions of years since life started on earth.

Actually succession and evolution would have been parallel processes at that time.
Succession is hence a process that starts where no living organisms are there – these could be
areas where no living organisms ever existed, say bare rock; or in areas that somehow, lost
all the living organisms that existed there. The former is called primary succession, while the
latter is termed secondary succession.

Pioneer species:

The species that invade a bare area are called pioneer species. In primary succession on
rocks these are usually lichens which are able to secrete acids to dissolve rock, helping in
weathering and soil formation. These later pave way to some very small plants like
bryophytes, which are able to take hold in the small amount of soil. They are, with time,
succeeded by bigger plants, and after several more stages, ultimately a stable climax forest
community is formed. The climax community remains stable as long as the environment
remains unchanged. With time the xerophytic habitat gets converted into a mesophytic one.

4. Solution: a)

Self-explanatory. As the level of prosperity increases among nations; an increase in


awareness and a desire to have lesser off-springs leads to a lower growth rate of population.

5. Solution: c)

Narrow top means high death rates as the base is broad (which means the birth rates are
high). High death rate and high birth rate can be easily correlated with the demography of a
developing country. It leads to an increase in the population.

6. Solution: c)

The two major sites of the Indus Valley Civilisation, along the Indus river—the cities of
Harappa in the north and Mohenjodaro in the south—showcase one of earliest examples of
civic planning. Other markers were houses, markets, storage facilities, offices, public baths,
etc., arranged in a grid-like pattern. There was also a highly developed drainage system.

http://insightsonindia.com INSIGHTS Page 2


Insights Mock tests – 2015: test 9 Solutions

While Harappa and Mohenjodaro are situated in Pakistan, the important sites excavated in
India are Lothal and Dholavira in Gujarat, Rakhigarhi in Haryana, Ropar in the Punjab,
Kalibangan and Balathal in Rajasthan, etc.

7. Solution: d)

The division of population into rural and urban is based on the residence. This division is
necessary because rural and urban life styles differ from each other in terms of their
livelihood and social conditions. The age-sex-occupational structure, density of population
(higher in urban) and level of development vary between rural and urban areas.

The criteria for differentiating rural and urban population varies from country to country. In
general terms rural areas are those where people are engaged in primary activities and
urban areas are those when majority of the working population is engaged in non-primary
activities.

8. Solution: d)

Just as any building is supported by pillars, the idea of human development is supported by
the concepts of equity, sustainability, productivity and empowerment.

Equity refers to making equal access to opportunities available to everybody. The


opportunities available to people must be equal irrespective of their gender, race, income
and in the Indian case, caste.

Sustainability means continuity in the availability of opportunities. To have sustainable


human development, each generation must have the same opportunities.

Empowerment means to have the power to make choices. Such power comes from
increasing freedom and capability. Good governance and people-oriented policies are
required to empower people. The empowerment of socially and economically
disadvantaged groups is of special importance.

9. Solution: d)

The Lion Capital discovered more than a hundred years ago at Sarnath, near Varanasi, is
generally referred to as Sarnath Lion Capital. This is one of the finest examples of sculpture
from the Mauryan period. Built in commemoration of the historical event of the first sermon
or the Dhammachakrapravartana by the Buddha at Sarnath, the capital was built by Ashoka.
The capital originally consisted of five component parts: (i) the shaft (which is broken in
many parts now), (ii) a lotus bell base, (iii) a drum on the bell base with four animals
proceeding clockwise, (iv) the figures of four majestic addorsed lions, and (v) the crowning
element, Dharamchakra, a large wheel, was also a part of this pillar. However, this wheel is
lying in a broken condition and is displayed in the site museum at Sarnath. The capital
without the crowning wheel and the lotus base has been adopted as the National Emblem of
Independent India.

http://insightsonindia.com INSIGHTS Page 3


Insights Mock tests – 2015: test 9 Solutions

10. Solution: d)

http://www.gnhbhutan.org/about/

11. Solution: d)

Refer to the section JUDICIARY AND PARLIAMENT – 11the NCERT Polity for a detailed
discussion on the Basic Structure Doctrine and its evolution.

12. Solution: c)

http://en.wikipedia.org/wiki/Carrying_capacity

Below carrying capacity, populations typically increase, while above, they typically
decrease. A factor that keeps population size at equilibrium is known as a regulating factor.
Population size decreases above carrying capacity due to a range of factors depending on
thespecies concerned, but can include insufficient space, food supply, or sunlight. The
carrying capacity of an environment may vary for different species and may change over
time due to a variety of factors, including: food availability, water supply, environmental
conditions and living space.

http://science.howstuffworks.com/environmental/green-science/earth-carrying-
capacity1.htm

http://insightsonindia.com INSIGHTS Page 4


Insights Mock tests – 2015: test 9 Solutions

13. Solution: a)

As many as 32 countries record low levels of human development. A large proportion of


these are small countries which have been going through political turmoil and social
instability in the form of civil war, famine or a high incidence of diseases. There is an urgent
need to address the human development requirements of this group through well thought
out policies.

International comparisons of human development can show some very interesting results.
Often people tend to blame low levels of human development on the culture of the people.
For example, X country has lower human development because its people follow Y religion,
or belong to Z community. Such statements are misleading.

To understand why a particular region keeps reporting low or high levels of human
development it is important to look at the pattern of government expenditure on the social
sector. The political environment of the country and the amount of freedom people have is
also important. Countries with high levels of human development invest more in the social
sectors and are generally free from political turmoil and instability. Distribution of the
country‘s resources is also far more equitable.

14. Solution: d)

Mixed Farming is found in the highly developed parts of the world, e.g. North-western
Europe, Eastern North America, parts of Eurasia and the temperate latitudes of Southern
continents.

Mixed farms are moderate in size and usually the crops associated with it are wheat, barley,
oats, rye, maize, fodder and root crops.

Fodder crops are an important component of mixed farming. Crop rotation and
intercropping play an important role in maintaining soil fertility. Equal emphasis is laid on
crop cultivation and animal husbandry. Animals like cattle, sheep, pigs and poultry provide
the main income along with crops.

Mixed farming is characterised by high capital expenditure on farm machinery and


building, extensive use of chemical fertilisers and green manures and also by the skill and
expertise of the farmers.

15. Solution: b)

Truck farming is a horticultural practice of growing one or more vegetable crops on a large
scale for shipment to distant markets. It is usually less intensive and diversified than market
gardening. At first this type of farming depended entirely on local or regional markets. As
the use of railroads and large-capacity trucks expanded and refrigerated carriers were
introduce.

http://insightsonindia.com INSIGHTS Page 5


Insights Mock tests – 2015: test 9 Solutions

16. Solution: d)

The first century CE onwards, Gandhara (now in Pakistan), Mathura in northern India and
Vengi in Andhra Pradesh emerged as important centres of art production. Buddha in the
symbolic form got a human form in Mathura and Gandhara. The sculptural tradition in
Gandhara had the confluence of Bactria, Parthia and the local Gandhara tradition. The local
sculptural tradition at Mathura became so strong that the tradition spread to other parts of
northern India. The best example in this regard is the stupa sculptures found at Sanghol in
the Punjab. The Buddha image at Mathura is modelled on the lines of earlier Yaksha images
whereas in Gandhara it has Hellenistic features.

Images of Vaishnava (mainly Vishnu and his various forms) and Shaiva (mainly the lingas
and mukhalingas) faiths are also found at Mathura but Buddhist images are found in large
numbers. It may be noted that the images of Vishnu and Shiva are represented by their
ayudhas (weapons). There is boldness in carving the large images, the volume of the images
is projected out of the picture plane, the faces are round and smiling, heaviness in the
sculptural volume is reduced to relaxed flesh. The garments of the body are clearly visible
and they cover the left shoulder. Images of the Buddha, Yakshas, Yakshinis, Shaivite and
Vaishnavite deities and portrait statues are profusely sculpted. In the second century CE,
images in Mathura get sensual, rotundity increases, they become fleshier. In the third
century CE, treatment of sculptural volume changes by reducing the extreme fleshiness,
movement in the posture is shown by increasing distance.

between the two legs as well as by using bents in the body posture. Softness in the surface
continues to get refined. The trend continues in the fourth century CE but in the late fourth
century CE, the massiveness and fleshiness is reduced further and the flesh becomes more
tightened, the volume of the drapery also gets reduced and in the fifth and sixth centuries
CE, the drapery is integrated into the sculptural mass. Transparent quality in the robes of
the Buddha images is evident. In this period, two important schools of sculptures in
northern India are worth noting. The traditional centre, Mathura, remained the main art
production site whereas Sarnath and Kosambi also emerged as important centres of art
production. Many Buddha images in Sarnath have plain transparent drapery covering both
shoulders, and the halo around the head has very little ornamentation whereas the Mathura
Buddha images continue to depict folds of the drapery in the Buddha images and the halo
around the head is profusely decorated. One can visit museums at Mathura, Sarnath,
Varanasi, New Delhi, Chennai, Amaravati, etc. to study the features of early sculptures.

17. Solution: a)

Decision-making

Cooperatives: Most cooperatives require simple-majority to pass a vote. That is, only over
50% of workers have to approve a motion for it to be passed.

Collectives: Collectives, however, use a consensus model: this means that every worker
must vote ―yes‖ in order to make a decision.

http://insightsonindia.com INSIGHTS Page 6


Insights Mock tests – 2015: test 9 Solutions

Hierarchies

While both cooperatives and collectives reject the hierarchical structure of typical
corporations (top-down, in which owners have authority over workers), the two take
different approaches to their structures.

Cooperatives: Every worker in a cooperative is also an owner. However, in some larger


cooperatives, there exists a form of hierarchy. This might mean that there is a president
(elected by the workers), a board of directors (also elected), and so on. However, all major
decisions are still made by votes taken in worker assemblies. Yet, most of the time, the
smaller the cooperative is the less formal its hierarchy will be – if it has one at all.

Collectives: Collectives tend to strive to abolish any form of hierarchy. This means that no
one worker in the collective has any more authority than another worker.

18. Solution: a)

Raw material used by industries should be cheap and easy to transport. Industries based on
cheap, bulky and weight-losing material (ores) are located close to the sources of raw
material such as steel, sugar, and cement industries. Perishability is a vital factor for the
industry to be located closer to the source of the raw material. Agro-processing and dairy
products are processed close to the sources of farm produce or milk supply respectively.

Processing cost will be the same whether it is processed at the site of off the site.

19. Solution: a)

Foot loose industries can be located in a wide variety of places. They are not dependent on
any specific raw material, weight losing or otherwise. They largely depend on component
parts which can be obtained anywhere. They produce in small quantity and also employ a
small labour force. These are generally not polluting industries. The important factor in their
location is accessibility by road network.

20. Solution: d)

Agro based Industries

Agro processing involves the processing of raw materials from the field and the farm into
finished products for rural and urban markets. Major agro-processing industries are food
processing, sugar, pickles, fruits juices, beverages (tea, coffee and cocoa), spices and oils fats
and textiles (cotton, jute, silk), rubber, etc.

Agri-business is commercial farming on an industrial scale often financed by business


whose main interests lie outside agriculture, for example, large corporations in tea
plantation business. Agri-business farms are mechanised, large in size, highly structured,
reliant on chemicals, and may be described as ‗agro-factories‘.

http://insightsonindia.com INSIGHTS Page 7


Insights Mock tests – 2015: test 9 Solutions

21. Solution: d)

The highest level of decision makers or policy makers perform quinary activities. These are
subtly different from the knowledge based industries that the quinary sector in general deals
with.

Quinary activities are services that focus on the creation, re-arrangement and interpretation
of new and existing ideas; data interpretation and the use and evaluation of new
technologies. Often referred to as ‗gold collar‘ professions, they represent another
subdivision of the tertiary sector representing special and highly paid skills of senior
business executives, government officials, research scientists, financial and legal consultants,
etc. Their importance in the structure of advanced economies far outweighs their numbers.

22. Solution: a)

Typhoid fever — also known simply as typhoid — is a common worldwide bacterial disease
transmitted by the ingestion of food or water contaminated with the feces of an infected
person, which contain the bacterium Salmonella enterica subsp. enterica,serovar Typhi.

The disease has received various names, such as gastric fever, enteric fever, abdominal
typhus, infantile remittant fever, slow fever, nervous fever and pythogenic fever. The
name typhoid means "resembling typhus" and comes from the neuropsychiatric symptoms
common to typhoid and typhus. Despite this similarity of their names, typhoid fever and
typhus are distinct diseases and are caused by different species of bacteria.

23. Solution: c)

Commensalism: This is the interaction in which one species benefits and the other is neither
harmed nor benefited. An orchid growing as an epiphyte on a mango branch, and barnacles
growing on the back of a whale benefit while neither the mango tree nor the whale derives
any apparent benefit. The cattle egret and grazing cattle in close association, a sight you are
most likely to catch if you live in farmed rural areas, is a classic example of commensalism.
The egrets always forage close to where the cattle are grazing because the cattle, as they
move, stir up and flush out from the vegetation insects that otherwise might be difficult for
the egrets to find and catch. Another example of commensalism is the interaction between
sea anemone that has stinging tentacles and the clown fish that lives among them. The fish
gets protection from predators which stay away from the stinging tentacles. The anemone
does not appear to derive any benefit by hosting the clown fish.

24. Solution: b)

Outsourcing or contracting out is giving work to an outside agency to improve efficiency


and reduce costs. When outsourcing involves transferring work to overseas locations, it is
described by the term off - shoring, although both off - shoring and outsourcing are used
together. Business activities that are outsourced include information technology (IT), human
resources, customer support and call centre services and at times also manufacturing and
engineering.

http://insightsonindia.com INSIGHTS Page 8


Insights Mock tests – 2015: test 9 Solutions

Data processing is an IT related service easily be carried out in Asian, East European and
African countries, In these countries IT skilled staff with good English language skills are
available at lower wages than those in the developed countries. Thus, a company in
Hyderabad or Manila does work on a project based on GIS techniques for a country like
U.S.A or Japan. Overhead costs are also much lower making it profitable to get job-work
carried out overseas, whether it is in India, China or even a less populous country like
Botswana in Africa.

25. Solution: a)

In Russia, a dense highway network is developed in the industrialised region west of the
Urals with Moscow as the hub. The important Moscow-Vladivostok Highway serves the
region to the east. Due to the vast geographical area, highways in Russia are not as
important as railways.

26. Solution: a)

The Mediterranean–Indian Ocean Route is a trade route that connects the highly
industrialised Western European region with West Africa, South Africa, South-east Asia and
the commercial agriculture and livestock economies of Australia and New Zealand. Before
the construction of the Suez Canal this was the route connecting Liverpool and Colombo
which was 6,400 km longer than the Suez Canal route. The volume of trade and traffic
between both East and West Africa is on the increase due to the development of the rich
natural resources such as gold, diamond, copper, tin, groundnut, oil palm, coffee and fruits.

27. Solution: d)

http://en.wikipedia.org/wiki/Inland_waterways_of_India

The significance of rivers as inland waterways for domestic and international transport and
trade has been recognised throughout the developed world. Despite inherent limitations,
many rivers have been modified to enhance their navigability by dredging, stabilising river
banks, and building dams and barrages for regulating the flow of water.

28. Solution: a)

Silk Road

 More than 2,000 years ago trade passed through Silk Road between Europe and
China, through India

 New route to cover China to South Asia, Central Asia and Europe

 Infrastructural development to boost regional connectivity

 Maritime connectivity

 Ecological cooperation

http://insightsonindia.com INSIGHTS Page 9


Insights Mock tests – 2015: test 9 Solutions

 Establishing special economic zones and industrial parks in the areas connected by
Silk Road

 Routes - Yunnan (South western province China) - Myanmar - Kolkata - "Silk Road
economic belt." The BCIM will play a key role in this economic corridor.

 South-eastern Chinese province Fujian to Chennai through littoral countries in the


region - Maritime Silk Road

 Funding expected to come by BRICS New Development Bank and proposed Asian
Infrastructural Investment bank

29. Solution: a)

International trade is the result of specialisation in production. It benefits the world


economy if different countries practise specialisation and division of labour in the
production of commodities or provision of services. Each kind of specialisation can give rise
to trade. Thus, international trade is based on the principle of comparative advantage,
complimentarity and transferability of goods and services and in principle, should be
mutually beneficial to the trading partners. In modern times, trade is the basis of the world‘s
economic organisation and is related to the foreign policy of nations. With well developed
transportation and communication systems, no country is willing to forego the benefits
derived from participation in international trade.

The principle of Sovereignty is in fact eroded by international trade.

30. Solution: d)

Look for the explanation in the next question.

31. Solution: d)

Age structure represents the number of people of different age groups. This is an important
indicator of population composition, since a large size of population in the age group of 15-
59 indicates a large working population. A greater proportion of population above 60
years represents an ageing population which requires more expenditure on health care
facilities. Similarly high proportion of young population would mean that the region has a
high birth rate and the population is youthful.

32. Solution: a)

The basic form of the Hindu temple comprises the following: (i) a cave-like sanctum
(garbhagriha literally ‗womb-house‘), which, in the early temples, was a small cubicle with a
single entrance and grew into a larger chamber in time. The garbhagriha is made to house
the main icon which is itself the focus of much ritual attention; (ii) the entrance to the temple
which may be a portico or colonnaded hall that incorporates space for a large number of
worshippers and is known as a mandapa; (iii) from the fifth century CE onwards,
freestanding temples tend to have a mountain- like spire, which can take the shape of a

http://insightsonindia.com INSIGHTS Page 10


Insights Mock tests – 2015: test 9 Solutions

curving shikhar in North India and a pyramidal tower, called a vimana, in South India; (iv)
the vahan, i.e., the mount or vehicle of the temple‘s main deity along with a standard pillar
or dhvaj is placed axially before the sanctum. Two broad orders of temples in the country
are known— Nagara in the north and Dravida in the south. At times, the Vesar style of
temples as an independent style created through the selective mixing of the Nagara and
Dravida orders is mentioned by some scholars.

33. Solution: c)

34. Solution: d)

Population ageing is the process by which the share of the older population becomes
proportionally larger. This is a new phenomenon of the twentieth century. In most of the
developed countries of the world, population in higher age groups has increased due to
increased life expectancy. With a reduction in birth rates, the proportion of children in the
population has declined.

35. Solution: a)

Most-favoured-nation (MFN): treating other people equally - Under the WTO agreements,
countries cannot normally discriminate between their trading partners. Grant someone a

http://insightsonindia.com INSIGHTS Page 11


Insights Mock tests – 2015: test 9 Solutions

special favour (such as a lower customs duty rate for one of their products) and you have to
do the same for all other WTO members.

This principle is known as most-favoured-nation (MFN) treatment. It is so important that it


is the first article of the General Agreement on Tariffs and Trade (GATT), which governs
trade in goods. MFN is also a priority in the General Agreement on Trade in Services
(GATS) (Article 2) and the Agreement on Trade-Related Aspects of Intellectual Property
Rights (TRIPS) (Article 4), although in each agreement the principle is handled slightly
differently. Together, those three agreements cover all three main areas of trade handled by
the WTO.

Some exceptions are allowed. For example, countries can set up a free trade agreement that
applies only to goods traded within the group — discriminating against goods from
outside. Or they can give developing countries special access to their markets. Or a country
can raise barriers against products that are considered to be traded unfairly from specific
countries. And in services, countries are allowed, in limited circumstances, to discriminate.
But the agreements only permit these exceptions under strict conditions. In general, MFN
means that every time a country lowers a trade barrier or opens up a market, it has to do so
for the same goods or services from all its trading partners — whether rich or poor, weak or
strong.

36. Solution: c)

The practice of selling a commodity in two countries at a price that differs for reasons not
related to costs is called dumping.

In international trade, the export by a country or company of a product at a price that is


lower in the foreign market than the price charged in the domestic market. As dumping
usually involves substantial export volumes of the product, it often has the effect of
endangering the financial viability of manufacturers or producers of the product in the
importing nation.

37. Solution: d)

WTO is the only international organisation dealing with the global rules of trade between
nations. It sets the rules for the global trading system and resolves disputes between its
member nations. WTO also covers trade in services, such as telecommunication and
banking, and others issues such as intellectual rights.

The WTO has however been criticised and opposed by those who are worried about the
effects of free trade and economic globalisation.

38. Solution: a)

World Health Organisation (WHO) suggests that, among other things, a ‗healthy city‘ must
have:

 A ‘Clean‘ and ‗Safe‘ environment.

http://insightsonindia.com INSIGHTS Page 12


Insights Mock tests – 2015: test 9 Solutions

 Meets the ‗Basic Needs‘ of ‗All‘ its inhabitants.


 Involves the ‗Community‘ in local government.
 Provides easily accessible ‗Health‘ service
39. Solution: b)

It's the acid in the ant's bite that cause the itch and sometimes more severe reactions.
Applying an antacid neutralizes the harmful acid effects.

Over production of acid in the stomach causes irritation and pain. In severe cases, ulcers are
developed in the stomach. Until 1970, only treatment for acidity was administration of
antacids, such as sodium hydrogencarbonate or a mixture of aluminium and magnesium
hydroxide. However, excessive hydrogencarbonate can make the stomach alkaline and
trigger the production of even more acid. Metal hydroxides are better alternatives because of
being insoluble, these do not increase the pH above neutrality. These treatments control only
symptoms, and not the cause. Therefore, with these metal salts, the patients cannot be
treated easily. In advanced stages, ulcers become life threatening and its only treatment is
removal of the affected part of the stomach.

40. Solution: a)

Histamine is a potent vasodilator. It has various functions. It contracts the smooth muscles
in the bronchi and gut and relaxes other muscles, such as those in the walls of fine blood
vessels. Histamine is also responsible for the nasal congestion associated with common cold
and allergic response to pollen. Synthetic drugs, brompheniramine (Dimetapp) and
terfenadine (Seldane), act as antihistamines. They interfere with the natural action of
histamine by competing with histamine for binding sites of receptor where histamine exerts
its effect.

41. Solution: d)

http://www.britannica.com/EBchecked/topic/602263/tranquilizer

42. Solution: d)

The chief instrument through which judicial activism has flourished in India is Public
Interest Litigation (PIL) or Social Action Litigation (SAL).

In normal course of law, an individual can approach the courts only if he/she has been
personally aggrieved. That is to say, a person whose rights have been violated, or who is
involved in a dispute, could move the court of law. This concept underwent a change
around 1979. In 1979, the Court set the trend when it decided to hear a case where the case
was filed not by the aggrieved persons but by others on their behalf. As this case involved a
consideration of an issue of public interest, it and such other cases came to be known as
public interest litigations. Around the same time, the Supreme Court also took up the case
about rights of prisoners.

This opened the gates for large number of cases where public spirited citizens and voluntary
organisations sought judicial intervention for protection of existing rights, betterment of life

http://insightsonindia.com INSIGHTS Page 13


Insights Mock tests – 2015: test 9 Solutions

conditions of the poor, protection of the environment, and many other issues in the interest
of the public. PIL has become the most important vehicle of judicial activism. Judiciary,
which is an institution that traditionally confined to responding to cases brought before it,
began considering many cases merely on the basis of newspaper reports and postal
complaints received by the court. Therefore, the term judicial activism became the more
popular description of the role of the judiciary.

43. Solution: b)

Chikungunya is an infection caused by the Chikungunya virus. It features sudden onset


fever usually lasting two to seven days, and joint pains typically lasting weeks or months
but sometimes years.

The virus is passed to humans by two species of mosquito of the genus Aedes: A.
albopictus and A. aegypti. The strain of chikungunya spreading to the US from the Caribbean
is most easily spread by A. aegypti. Concern exists that this strain of chikungunya could
mutate to make the A. albopictus vector more efficient. If this mutation were to occur,
chikungunya would be more of a public health concern to the US because the A. albopictus or
Asian tiger mosquito is more widespread in the US and is more aggressive than the A.
aegypti. Animal reservoirs of the virus include monkeys, birds, cattle, and rodents. This is in
contrast to dengue, for which only humans and primates are hosts.

44. Solution: d)

Analgesics reduce or abolish pain without causing impairment of consciousness, mental


confusion, incoordination or paralysis or some other disturbances of nervous system. These
are classified as follows:

(i) Non-narcotic (non-addictive) analgesics

(ii) Narcotic drugs

(i) Non-narcotic (non-addictive) analgesics: Aspirin and paracetamol belong to the class of non-
narcotic analgesics. Aspirin is the most familiar example. Aspirin inhibits the synthesis of
chemicals known as prostaglandins which stimulate inflammation in the tissue and cause
pain.

(ii) Narcotic analgesics: Morphine and many of its homologues, when administered in
medicinal doses, relieve pain and produce sleep. In poisonous doses, these produce stupor,
coma, convulsions and ultimately death. Morphine narcotics are sometimes referred to as
opiates, since they are obtained from the opium poppy. These analgesics are chiefly used for
the relief of postoperative pain, cardiac pain and pains of terminal cancer, and in child birth.

45. Solution: a)

http://en.wikipedia.org/wiki/Antibiotic_resistance#Cause

http://insightsonindia.com INSIGHTS Page 14


Insights Mock tests – 2015: test 9 Solutions

Natural selection is basically the Darwinian Principle. The bacteria that do not die after the
application of anti-biotics breed among themselves and form huge colonies later.

46. Solution: c)

http://www.cbd.int/abs/

http://en.wikipedia.org/wiki/Bioprospecting#Convention_on_Biological_Diversity_.28CB
D.29

Biopiracy is the term used to refer to the use of bio-resources by multinational companies
and other organisations without proper authorisation from the countries and people
concerned without compensatory payment. Most of the industrialised nations are rich
financially but poor in biodiversity and traditional knowledge. In contrast the developing
and the underdeveloped world is rich in biodiversity and traditional knowledge related to
bio-resources. Traditional knowledge related to bio-resources can be exploited to develop
modern applications and can also be used to save time, effort and expenditure during their
commercialisation.

47. Solution: a)

The removal of judges of the Supreme Court and the High Courts is also extremely difficult.
A judge of the Supreme Court or High Court can be removed only on the ground of proven
misbehaviour or incapacity. A motion containing the charges against the judge must be
approved by special majority in both Houses of the Parliament.

It is clear from this procedure that removal of a judge is a very difficult procedure and
unless there is a general consensus among Members of the Parliament, a judge cannot be
removed. It should also be noted that while in making appointments, the executive plays a
crucial role; the legislature has the powers of removal. This has ensured both balance of
power and independence of the judiciary. So far, only one case of removal of a judge of the
Supreme Court came up for consideration before the Parliament. In that case, though the
motion got two-thirds majority, it did not have the support of the majority of the total
strength of the House and therefore, the judge was not removed.

48. Solution: b)

Antiseptics and disinfectants are also the chemicals which either kill or prevent the growth
of microorganisms. Antiseptics are applied to the living tissues such as wounds, cuts, ulcers
and diseased skin surfaces. Examples are furacine, soframicine, etc. These are not ingested
like antibiotics. Commonly used antiseptic, dettol is a mixture of chloroxylenol and
terpineol. Bithionol (the compound is also called bithional) is added to soaps to impart
antiseptic properties. Iodine is a powerful antiseptic. Its 2-3 per cent solution in alcohol
water mixture is known as tincture of iodine. It is applied on wounds. Iodoform is also used
as an antiseptic for wounds. Boric acid in dilute aqueous solution is weak antiseptic for eyes.
Disinfectants are applied to inanimate objects such as floors, drainage system, instruments,
etc. Same substances can act as an antiseptic as well as disinfectant by varying the

http://insightsonindia.com INSIGHTS Page 15


Insights Mock tests – 2015: test 9 Solutions

concentration. For example, 0.2 per cent solution of phenol is an antiseptic while its one
percent solution is disinfectant. Chlorine in the concentration of 0.2 to 0.4 ppm in aqueous
solution and sulphur dioxide in very low concentrations, are disinfectants.

49. Solution: b)

Antibiotic revolution has provided long and healthy life to people. The life expectancy has
almost doubled. The increased population has caused many social problems in terms of food
resources, environmental issues, employment, etc. To control these problems, population is
required to be controlled. This has lead to the concept of family planning. Antifertility drugs
are of use in this direction. Birth control pills essentially contain a mixture of synthetic
estrogen and progesterone derivatives. Both of these compounds are hormones. It is known
that progesterone suppresses ovulation. Synthetic progesterone derivatives are more potent
than progesterone. Norethindrone is an example of synthetic progesterone derivative most
widely used as antifertility drug. The estrogen derivative which is used in combination with
progesterone derivative is ethynylestradiol (novestrol).

50. Solution: d)

International comparisons of human development are interesting. Size of the territory and
per capita income are not directly related to human development. Often smaller countries
have done better than larger ones in human development. Similarly, relatively poorer
nations have been ranked higher than richer neighbours in terms of human development.

For example, Sri Lanka, Trinidad and Tobago have a higher rank than India in the human
development index despite having smaller economies. Similarly, within India, Kerala
performs much better than Punjab and Gujarat in human development despite having lower
per capita income.

51. Solution: d)

Chemicals are added to food for (i) their preservation, (ii) enhancing their appeal, and (iii)
adding nutritive value in them. Main categories of food additives are as follows:

(i) Food colours

(ii) Flavours and sweeteners

(iii) Fat emulsifiers and stabilising agents

(iv) Flour improvers - antistaling agents and bleaches

(v) Antioxidants

(vi) Preservatives

(vii) Nutritional supplements such as minerals, vitamins and amino acids.

Except for chemicals of category (vii), none of the above additives have nutritive value.
These are added either to increase the shelf life of stored food or for cosmetic purposes.

http://insightsonindia.com INSIGHTS Page 16


Insights Mock tests – 2015: test 9 Solutions

52. Solution: d)

Federalism does not consist of a set of fixed principles, which are applied, to different
historical situations. Rather, federalism as a principle of government has evolved differently
in different situations. American federalism – one of the first major attempts to build a
federal polity – is different from German or Indian federalism. But there are also a few key
ideas and concepts associated with federalism.

 Essentially, federalism is an institutional mechanism to accommodate two sets of


polities—one at the regional level and the other at the national level. Each
government is autonomous in its own sphere. In some federal countries, there is
even a system of dual citizenship. India has only a single citizenship.
 The people likewise, have two sets of identities and loyalties—they belong to the
region as well as the nation, for example we are Gujaratis or Jharkhandis as well as
Indians. Each level of the polity has distinct powers and responsibilities and has a
separate system of government.
 The details of this dual system of government are generally spelt out in a written
constitution, which is considered to be supreme and which is also the source of the
power of both sets of government. Certain subjects, which concern the nation as a
whole, for example, defence or currency, are the responsibility of the union or central
government. Regional or local matters are the responsibility of the regional or State
government.
 To prevent conflicts between the centre and the State, there is an independent judiciary
to settle disputes. The judiciary has the powers to resolve disputes between the central
government and the States on legal matters about the division of power.
53. Solution: d)

Aspartame is the most successful and widely used artificial sweetener. It is roughly 100
times as sweet as cane sugar. It is methyl ester of dipeptide formed from aspartic acid and
phenylalanine. Use of aspartame is limited to cold foods and soft drinks because it is
unstable at cooking temperature.

Alitame is high potency sweetener, although it is more stable than aspartame, the control of
sweetness of food is difficult while using it.

Sucrolose is trichloro derivative of sucrose. Its appearance and taste are like sugar. It is
stable at cooking temperature. It does not provide calories.

54. Solution: d)

Food preservatives prevent spoilage of food due to microbial growth. The most commonly
used preservatives include table salt, sugar, vegetable oils and sodium benzoate,
C6H5COONa. Sodium benzoate is used in limited quantities and is metabolised in the body.
Salts of sorbic acid and propanoic acid are also used as preservatives.

http://insightsonindia.com INSIGHTS Page 17


Insights Mock tests – 2015: test 9 Solutions

55. Solution: d)

Basically all soaps are made by boiling fats or oils with suitable soluble hydroxide.
Variations are made by using different raw materials.

Toilet soaps are prepared by using better grades of fats and oils and care is taken to remove
excess alkali. Colour and perfumes are added to make these more attractive.

Soaps that float in water are made by beating tiny air bubbles before their hardening.
Transparent soaps are made by dissolving the soap in ethanol and then evaporating the excess
solvent.

In medicated soaps, substances of medicinal value are added. In some soaps, deodorants are
added. Shaving soaps contain glycerol to prevent rapid drying. A gum called, rosin is added
while making them.

It forms sodium rosinate which lathers well. Laundry soaps contain fillers like sodium
rosinate, sodium silicate, borax and sodium carbonate.

56. Solution: b)

The cutting of DNA by restriction endonucleases results in the fragmentes of DNA. These
fragments can be separated by a technique known as gel electrophoresis. Since DNA
fragments are negatively charged molecules they can be separated by forcing them to move
towards the anode under an electric field through a medium/matrix. Nowadays the most
commonly used matrix is agarose which is a natural polymer extracted from sea weeds. The
DNA fragments separate (resolve) according to their size through sieving effect provided by
the agarose gel. Hence, the smaller the fragment size, the farther it moves.

57. Solution: d)

The Constitution of India provides for a single integrated judicial system. This means that
unlike some other federal countries of the world, India does not have separate State courts.
The structure of the judiciary in India is pyramidal with the Supreme Court at the top, High
Courts below them and district and subordinate courts at the lowest level (see the diagram
below). The lower courts function under the direct superintendence of the higher courts.

http://insightsonindia.com INSIGHTS Page 18


Insights Mock tests – 2015: test 9 Solutions

58. Solution: b)

Hard water contains calcium and magnesium ions. These ions form insoluble calcium and
magnesium soaps respectively when sodium or potassium soaps are dissolved in hard
water.

These insoluble soaps separate as scum in water and are useless as cleansing agent. In fact
these are hinderance to good washing, because the precipitate adheres onto the fibre of the
cloth as gummy mass. Hair washed with hard water looks dull because of this sticky
precipitate. Dye does not absorb evenly on cloth washed with soap using hard water,
because of this gummy mass.

59. Solution: a)

The need to have a sustainable energy supply necessitates the exploration of available
energy resources. Among these, renewable energy resources are in the forefront. It is now an
established fact that renewable energy can be an integral part of sustainable development
because of its inexhaustible nature and environment friendly features. Renewable energy
can also play an important role in resolving the energy crisis in urban areas to a great
extend. So far , renewable energy projects with an aggregate capacity of about 33,200 MW
have been installed in the country which includes 22,168 MW of wind power, 2870 MW of
solar power, 4225 MW of bio power and 3938 MW of small hydro power.

http://insightsonindia.com INSIGHTS Page 19


Insights Mock tests – 2015: test 9 Solutions

http://pib.nic.in/newsite/efeatures.aspx

60. Solution: d)

The decision of the Government to approve the North Eastern Region Power System
Improvement Project (NERPSIP) is in tune with its commitment for the economic
development of north eastern States. As the Intra – State Transmission and Distribution
systems in the north eastern states have remained very weak; the Central Electricity
Authority developed a comprehensive scheme for the North East Region (NER) in
consultation with the Power Grid Corporation of India Limited (PGCIL).

It is for the six States of Assam, Manipur, Meghalaya, Mizoram, Tripura and Nagaland.

The main aim of the project is to address the lacuna in the Intra-State Transmission &
Distribution Infrastructure in the north east region. A similar scheme was earlier approved
for Arunachal Pradesh and Sikkim at an estimated cost of Rs 4754.42 crore.

The Scheme is to be taken up under a new Central Sector Plan Scheme of Ministry of Power
and will be implemented with the assistance of World Bank loan and by the Government of
India. The NERPSIP will be funded on 50:50 (World Bank: GoI) basis except the component
of capacity building for Rs 89 crore, which will be fully funded by the Government of India.
This project is the first phase of the three phased development, for which the World Bank
will be extending loan in three tranches of US $ 500 million each.

61. Solution: a)

is located a hundred kilometres from Ajanta and has thirty-two Buddhist, Brahmanical and
Jain caves. It is a unique art-historical site in the country as it has monastries associated with
the three religions dating from the fifth century CE onwards to the eleventh century CE.

It is also unique in terms of stylistic eclecticism, i.e., confluence of many styles at one place.
The caves of Ellora and Aurangabad show the ongoing differences between the two
religions—Buddhism and Brahmanical. There are twelve Buddhist caves having many
images belonging to Vajrayana Buddhism like Tara, Mahamayuri, Akshobhya,
Avalokiteshwara, Maitrya, Amitabha, etc. Buddhist caves are big in size and are of single,
double and triple storeys. Their pillars are massive. Ajanta also has excavated double-
storeyed caves but at Ellora, the triple storey is a unique achievement. All the caves were
plastered and painted but nothing visible is left. The shrine Buddha images are big in size;
they are generally guarded by the images of Padmapani and Vajrapani. Cave No. 12, which
is a triple-storey excavation, has images of Tara, Avalokiteshwara, Manushi Buddhas and
the images of Vairochana, Akshobhya, Ratnasambhava, Amitabha, Amoghsiddhi,
Vajrasatva and Vajraraja. On the other hand, the only double-storey cave of the Brahmanical
faith is Cave No. 14. Pillar designs grow from the Buddhist caves and when they reach the

http://insightsonindia.com INSIGHTS Page 20


Insights Mock tests – 2015: test 9 Solutions

Jain caves belonging to the ninth century CE, they become very ornate and the decorative
forms gain heavy protrusion.

62. Solution: d)

http://hdr.undp.org/en/content/human-development-index-hdi

There is a separate Gender Index to capture female labour participation rates.

63. Solution: d)

On the manufacturing front, the Government launched a ―Make in India‖ programme so


that the Indian products would get a leg-up in terms of being competitive both in price and
quality not only within the country but also abroad. In order to enable domestic
manufacturing to gain cost advantage in local production, the authorities took several steps
to improve the doing business ethos within the country through administrative actions so
that the pronounced pro-growth and pro-business approach is understood and used up by
entrepreneurial forces. In fact, the Union Finance Minister Mr. Jaitely has time and again
stated that pro-growth and pro-business approach does not stand in conflict with pro-poor
policies as the resultant gains accruing from higher growth would level the field by making
the government to increase its outlays on public programmes, including genuine welfare
measures that help poverty elimination. The government also opened up important sectors
like defense and railways for overseas investment and increasing the FDI cap in the
insurance segment and the foreign direct investment into these fields would definitely go a
long way in improving these sectors in terms of transfer of technology and best managerial
practices, besides bolstering their capital base for further expansion and modernization in
the short to medium-term. As manufacturing requires the motor of power, the government
swiftly took initiatives to improve the power situation for domestic users, both individuals
and industry.

Following a Supreme Court ruling invalidating the allocation of over 200 coal blocs since
1993 that left dozens of private companies in uncertainty over their power supplies, the
government came out with an Ordinance to sort out the mess on October 20. The
government can now reassign these blocs through a transparent, online auction than an
earlier practice that led to sordid happenings and allegations of corruption in allocations.
The latest action on the coal front would help the power, steel and cement companies that
rely on these coal supplies, besides benefitting the coal-rich States to get the revenue from
auctions directly instead through the Centre. The new Ordinance also contains an enabling
provision that recognizes the need to open up coal mining to private companies. All these
measures for revival of activities across a broad range of industries through swift
administrative action duly backed up by political class to secure the requisite seal of
approval from Parliament meant the restoration of the much-needed investors‘ confidence.

http://pib.nic.in/newsite/efeatures.aspx

http://insightsonindia.com INSIGHTS Page 21


Insights Mock tests – 2015: test 9 Solutions

64. Solution: d)

Key facts about Thatheras

 The craft of the Thatheras constitutes the traditional techniques of


manufacturing brass, copper and kansa (an alloy of copper, zinc and tin) utensils.

 They have a unique ethnic and historical identity with an oral tradition that
underpins their skill.

 The name of the community – ‗Thatheras‘ is identical with the name of the element.

 These craft utensils of Thatheras have both utilitarian and ritualistic value.

 The tradition of using the metals is recommended by the ancient Indian school of
medicine, Ayurveda.

 Skills of the Thatheras have been orally transmitted from one generation to other
generations.

 During the reign of Maharaja Ranjit Singh (1883) – the great 19thCentury Sikh
Monarch, the Thatheras crafts colony was established in Jandiala Guru, Punjab.
Thus, Jandiala Guru became an area of repute due to the skill of the Thatheras.

65. Solution: a)

Beijing Protocol: This Protocol has brought out new principal offences in hijacking. This
protocol is combined with ancillary offences, enlarged the scope of ‗hijacking‘, expanded
jurisdiction and strengthened extradition and mutual assistance regimes in case hijacking.

The government has recently given nod to for introduction of the comprehensive Anti-
Hijacking Bill, 2014 in the Parliament. The Cabinet has also given its approval to
ratify Beijing Protocol, 2010 of the UN body International Civil Aviation Organisation
(ICAO) to which India is a signatory.

66. Solution: b)

Acquired immunity, on the other hand, is pathogen specific. It is characterised by memory.


This means that our body when it encounters a pathogen for the first time produces a
response called primary response which is of low intensity. Subsequent encounter with the
same pathogen elicits a highly intensified secondary or anamnestic response. This is
ascribed to the fact that our body appears to have memory of the first encounter.

The primary and secondary immune responses are carried out with the help of two special
types of lymphocytes present in our blood, i.e., B-lymphocytes and T-lymphocytes. The B-
lymphocytes produce an army of proteins in response to pathogens into our blood to fight
with them. These proteins are called antibodies. The T-cells themselves do not secrete
antibodies but help B cells produce them.

http://insightsonindia.com INSIGHTS Page 22


Insights Mock tests – 2015: test 9 Solutions

67. Solution: c)

In the year 1963, the two enzymes responsible for restricting the growth of bacteriophage in
Escherichia coli were isolated. One of these added methyl groups to DNA, while the other
cut DNA. The later was called restriction endonuclease. The first restriction endonuclease–
Hind II, whose functioning depended on a specific DNA nucleotide sequence was isolated
and characterised five years later. It was found that Hind II always cut DNA molecules at a
particular point by recognising a specific sequence of six base pairs. This specific base
sequence is known as the recognition sequence for Hind II. Besides Hind II, today we know
more than 900 restriction enzymes that have been isolated from over 230 strains of bacteria
each of which recognise different recognition sequences.

68. Solution: c)

A Parliamentary Standing Committee on External Affairs has asked the government to


present Constitution (119th Amendment) Bill, 2013 without any delay to give effect to
the India-Bangladesh Land Boundary Agreement (LBA).

The Constitution (119th Amendment) Bill, 2013 aims to ratify the LBA between India and
Bangladesh under the Indira-Mujib pact of 1974 to exchange areas and people on either side
of the border.

As per the provision of pact, India will exchange 111 enclaves measuring 17,160 acres with
Bangladesh and receive 51 enclaves covering 7,110 acres. Almost 51,000 people reside in
these enclaves. The territories involved in the exchange are in Assam, West
Bengal, Meghalaya and Tripura.

On 16th May 1974, soon after the independence of Bangladesh the LBA was signed between
India and Bangladesh in order to find a solution to the complex nature of border
demarcation. This pact was ratified by Bangladesh government in 1974.

In 1974, in India Cabinet had granted approval to this pact but was not ratified as it involved
cession of territory.

69. Solution: a)

PM Modi by wearing a traditional Naga headgear and by beating the traditional Naga gong
three times declared the festival open at the picturesque heritage village of Kisama, Kohima.

About Hornbill festival of Nagaland

Hornbill (name of a bird) Festival is celebrated in Nagaland every year in the first week of
December. This festival is 10-day tourism promotional extravaganza which showcases
rich culture of Naga tribes which includes shows of music and dance.

It was established on 1st December 1963 and was inaugurated by the then President Dr. S
Radhakrishnan. The festival is organized by the State Tourism and Art & Culture
Departments, at Naga Heritage Village, Kisama which is about 12 km from Kohima the state

http://insightsonindia.com INSIGHTS Page 23


Insights Mock tests – 2015: test 9 Solutions

capital. Notably, it is also supported by the Centre as one of the biggest


indigenous festivals of the country.

The Hornbill Festival provides a colourful mixture of dances, performances, crafts, parades,
games, sports, food fairs and religious ceremonies. The festival both exposes the culture and
tradition of tribal peoples, and reinforces Nagaland‘s identity as a unique state in India‘s
federal union.

70. Solution: a)

Hemp: It is the common name for plants of the entire cannabis. This term is often used to
refer only to cannabis strains cultivated for industrial (non-drug) use.

Industrial hemp: It has many uses, including paper, textiles, biodegradable plastics,
construction, health food, fuel and hemp oil, wax, resin, rope and pulp.

Implications of hemp industry: Industrial hemp can bring a revolution in the industry as
hemp fibre is the strongest one in the world. It has the potential to cure cancer and a next
generation crop to bring revolution to our Indian economy.

71. Solution: d)

It is an integrated bill payment system offering inter-operable and accessible bill payment
service to customers through a network of agents, enabling multiple payment modes, and
providing instant confirmation of payment.

About BBPS

 It will help consumers pay multiple bills like electricity, telephone and school fees at
a single point of transaction.

 The National Payment Corporation of India (NPCI) has been appointed as the nodal
body which will set the standards, and also take care of clearing and settlement as
the Bharat Bill Payment Central Unit (BBPCU).

 In the two tiered BBPS set-up, there will be authorised operational units called
Bharat Bill Payment Operating Units (BBPOUs) with an agent network under the
BBPCU.

 RBI has set a Rs 100-crore networth and domestic registration as qualifying


conditions for those seeking to be authorised collection agents.

 Participants in the BBPS will include authorised entities such as BPCU, BBPOUs as
well as their authorised agents, payment gateways, banks, billers and service
providers, and other entities, including authorised prepaid payment instrument
issuers.

Thus, BBPS will help track all the payments being made in economy, including cash
payments to utilities, schools, and telcos among others.

http://insightsonindia.com INSIGHTS Page 24


Insights Mock tests – 2015: test 9 Solutions

72. Solution: d)

Mount Aso, a volcano in southern Japan has erupted. It is the first such eruption in 22 years,
causing flight cancellations and prompting warnings to stay away from its crater. This
eruption has spewed out lava debris and smoke, shooting plumes of ash a kilometre into the
sky.

Earlier, Japan Meteorological Agency (JMA) had reported earthquakes and other seismic
activity in late August on Mount Aso.

About Mount Aso

Mount Aso is the largest active volcano in Japan, and is among the largest in the world. It is
located in Aso Kuju National Park in Kumamoto Prefecture, on Kyushu Island. Its peak is
1592 m above sea level. Mount Aso has one of the largest calderas in the world (25 km north-
south and 18 km east-west). The caldera has a circumference of around 120 km.

73. Solution: a)

 The bill includes certain communities in the list of Scheduled Castes


from Kerala, Madhya Pradesh, Odisha, and Tripura. They are

1. Kerala: Pulluvan, Thachar (other than ‗Carpenter‘).

2. Odisha: Amata, Amath, Bajia, Jaggili, Jagli, Buna Pano.

3. Tripura: Chamar-Rohidas, Chamar-Ravidas, Dhobi, Jhalo-Malo.

 This amendment Bill removes a Majhi (Nepali) community from the list of Scheduled
Castes in Sikkim.

74. Solution: d)

The Trade Facilitation Agreement (TFA) is the WTO‘s first-ever multilateral accord. It was
outcome of 9th Ministerial Conference of WTO for broader reform to boost global trade held
in Bali, Indonesia in December, 2013.

The agreement includes provisions for

 Lowering import tariffs and agricultural subsidies: It will make it easier for
developing countries to trade with the developed world in global markets.

 Abolish hard import quotas: Developed countries would abolish hard import quotas
on agricultural products from the developing world and instead would only be
allowed to charge tariffs on amount of agricultural imports exceeding specific limits.

 Reduction in red tape at international borders: It aims to reduce red-tapism to


facilitate trade by reforming customs bureaucracies and formalities.

Implications

http://insightsonindia.com INSIGHTS Page 25


Insights Mock tests – 2015: test 9 Solutions

If this agreement is properly implemented, it may create US$1 trillion worth of global
economic activity which may add 21 million new jobs and lower the cost of doing
international trade by 10–15 percent.

75. Solution: d)

Perhaps the most important power of the Supreme Court is the power of judicial review.
Judicial Review means the power of the Supreme Court (or High Courts) to examine the
constitutionality of any law if the Court arrives at the conclusion that the law is inconsistent
with the provisions of the Constitution, such a law is declared as unconstitutional and
inapplicable. The term judicial review is nowhere mentioned in the Constitution. However,
the fact that India has a written constitution and the Supreme Court can strike down a law
that goes against fundamental rights, implicitly gives the Supreme Court the power of
judicial review.

In the case of federal relations too, the Supreme Court can use the review powers if a law is
inconsistent with the distribution of powers laid down by the Constitution. Suppose, the
central government makes a law, which according to some States, concerns a subject from
the State list- then the States can go to the Supreme Court and if the court agrees with them,
it would declare that the law is unconstitutional. In this sense, the review power of the
Supreme Court includes power to review legislations on the ground that they violate
fundamental rights or on the ground that they violate the federal distribution of powers. The
review power extends to the laws passed by State legislatures also.

Together, the writ powers and the review power of the Court make judiciary very powerful.
In particular, the review power means that the judiciary can interpret the Constitution and
the laws passed by the legislature.

76. Solution: c)

A plasmid is a small DNA molecule within a cell that is physically separated from a
chromosomal DNA and can replicate independently. They are most commonly found in
bacteria as small, circular, double-stranded DNA molecules, however plasmids are
sometimes present in archaea and eukaryotic organisms. In nature, plasmids carry genes
that may benefit survival of the organism (e.g. antibiotic resistance), and can frequently be
transmitted from one bacterium to another (even of another species) via horizontal gene
transfer. While the chromosomes are big and contain all the essential information for living
(an adequate analogy is the hard-drive of a computer), plasmids usually are very small and
contain additional information (in this analogy, plasmids are the USB flash drives). Artificial
plasmids are widely used as vectors in molecular cloning, serving to drive the replication of
recombinant DNA sequences within host organisms.

77. Solution: d)

http://www.downtoearth.org.in/content/invasion-triggers-evolution

http://insightsonindia.com INSIGHTS Page 26


Insights Mock tests – 2015: test 9 Solutions

78. Solution: d)

http://www.downtoearth.org.in/content/varied-visions-new-climate-agreement-lima

79. Solution: d)

http://www.downtoearth.org.in/content/watershed-conservation-can-benefit-700-million-
people-100-big-cities

80. Solution: d)

http://www.downtoearth.org.in/content/supreme-court-sets-social-justice-bench-deal-
issues

81. Solution: c)

http://www.downtoearth.org.in/content/japan-uses-climate-finance-lend-1-billion-
indonesia-dirty-coal

http://www.theguardian.com/world/2014/sep/17/chinas-ban-on-dirty-coal-could-cost-
australian-mining-almost-15bn

82. Solution: b)

http://www.downtoearth.org.in/content/india-get-25-solar-parks

83. Solution: c)

http://www.downtoearth.org.in/content/274-tiger-deaths-four-years-india-records-
highest-toll-between-two-censuses

84. Solution: d)

The rock-cut cave carved at Barabar hills near Gaya in Bihar is known as the Lomus Rishi
cave. The facade of the cave is decorated with the semicircular chaitya arch as the entrance.
The elephant frieze carved in high relief on the chaitya arch shows considerable movement.
The interior hall of this cave is rectangular with a circular chamber at the back. The entrance
is located on the side wall of the hall. The cave was patronised by Ashoka for the Ajivika
sect. The Lomus Rishi cave is an isolated example of this period. But many Buddhist caves of
the subsequent periods were excavated in eastern and western India.

Due to the popularity of Buddhism and Jainism, stupas and viharas were constructed on a
large scale. However, there are also examples of a few Brahmanical gods in the sculptural
representations. It is important to note that the stupas were constructed over the relics of the
Buddha at Rajagraha, Vaishali, Kapilavastu, Allakappa, Ramagrama, Vethadipa, Pava,
Kushinagar and Pippalvina. The textual tradition also mentions construction of various
other stupas on the relics of the Buddha at several places including Avanti and Gandhara
which are outside the Gangetic valley.

http://insightsonindia.com INSIGHTS Page 27


Insights Mock tests – 2015: test 9 Solutions

85. Solution: d)

http://www.downtoearth.org.in/content/geoengineering-could-be-catastrophic-planet

http://en.wikipedia.org/wiki/Climate_engineering

86. Solution: d)

In any society, disputes are bound to arise between individuals, between groups and
between individuals or groups and government.

All such disputes must be settled by an independent body in accordance with the principle
of rule of law. This idea of rule of law implies that all individuals — rich and poor, men or
women, forward or backward castes — are subjected to the same law. The principal role of
the judiciary is to protect rule of law and ensure supremacy of law. It safeguards rights of
the individual, settles disputes in accordance with the law and ensures that democracy does
not give way to individual or group dictatorship. In order to be able to do all this, it is
necessary that the judiciary is independent of any political pressures.

87. Solution: c)

BOD refers to the amount of the oxygen that would be consumed if all the organic matter in
one liter of water were oxidised by bacteria. The sewage water is treated till the BOD is
reduced. The BOD test measures the rate of uptake of oxygen by micro-organisms in a
sample of water and thus, indirectly, BOD is a measure of the organic matter present in the
water. The greater the BOD of waste water, more is its polluting potential.

88. Solution: b)

Simply stated independence of judiciary means that

 the other organs of the government like the executive and legislature must not
restrain the functioning of the judiciary in such a way that it is unable to do justice.
 the other organs of the government should not interfere with the decision of the
judiciary.
 judges must be able to perform their functions without fear or favour.

Independence of the judiciary does not imply arbitrariness or absence of accountability.


Judiciary is a part of the democratic political structure of the country. It is therefore
accountable to the Constitution, to the democratic traditions and to the people of the
country.

89. Solution: d)

The judges have a fixed tenure. They hold office till reaching the age of retirement. Only in
exceptional cases, judges may be removed. But otherwise, they have security of tenure.
Security of tenure ensures that judges could function without fear or favour. The
Constitution prescribes a very difficult procedure for removal of judges.

http://insightsonindia.com INSIGHTS Page 28


Insights Mock tests – 2015: test 9 Solutions

The Constitution makers believed that a difficult procedure of removal would provide
security of office to the members of judiciary.

The judiciary is not financially dependent on either the executive or legislature. The
Constitution provides that the salaries and allowances of the judges are not subjected to the
approval of the legislature. The actions and decisions of the judges are immune from
personal criticisms. The judiciary has the power to penalise those who are found guilty of
contempt of court. This authority of the court is seen as an effective protection to the judges
from unfair criticism. Parliament cannot discuss the conduct of the judges except when the
proceeding to remove a judge is being carried out. This gives the judiciary independence to
adjudicate without fear of being criticised.

90. Solution: b)

Archaeologists have discovered thousands of seals, usually made of steatite, and


occasionally of agate, chert, copper, faience and terracotta, with beautiful figures of animals,
such as unicorn bull, rhinoceros, tiger, elephant, bison, goat, buffalo, etc. The realistic
rendering of these animals in various moods is remarkable. The purpose of producing seals
was mainly commercial. It appears that the seals were also used as amulets, carried on the
persons of their owners, perhaps as modern-day identity cards. The standard Harappan seal
was a square plaque 2×2 square inches, usually made from the soft river stone, steatite.
Every seal is engraved in a pictographic script which is yet to be deciphered. Some seals
have also been found in gold and ivory. They all bear a great variety of motifs, most often of
animals including those of the bull, with or without the hump, the elephant, tiger, goat and
also monsters. Sometimes trees or human figures were also depicted.

The most remarkable seal is the one depicted with a figure in the centre and animals around.
This seal is generally identified as the Pashupati Seal by some scholars whereas some
identify it as the female deity. This seal depicts a human figure seated cross-legged. An
elephant and a tiger are depicted to the right side of the seated figure, while on the left a
rhinoceros and a buffalo are seen. In addition to these animals two antelopes are shown
below the seat. Seals such as these date from between 2500 and 1500 BCE and were found in
considerable numbers in sites such as the ancient city of Mohenjodaro in the Indus Valley.
Figures and animals are carved in intaglio on their surfaces. Square or rectangular copper
tablets, with an animal or a human figure on one side and an inscription on the other, or an
inscription on both sides have also been found. The figures and signs are carefully cut with a
burin. These copper tablets appear to have been amulets. Unlike inscriptions on seals which
vary in each case, inscriptions on the copper tablets seem to be associated with the animals
portrayed on them.

91. Solution: c)

Original jurisdiction means cases that can be directly considered by the Supreme Court
without going to the lower courts before that.

Form the diagram below, you will notice that cases involving federal relations go directly to
the Supreme Court. The Original Jurisdiction of the Supreme Court establishes it as an

http://insightsonindia.com INSIGHTS Page 29


Insights Mock tests – 2015: test 9 Solutions

umpire in all disputes regarding federal matters. In any federal country, legal disputes are
bound to arise between the Union and the States; and among the States themselves. The
power to resolve such cases is entrusted to the Supreme Court of India. It is called original
jurisdiction because the Supreme Court alone has the power to deal with such cases. Neither
the High Courts nor the lower courts can deal with such cases. In this capacity, the Supreme
Court not just settles disputes but also interprets the powers of Union and State government
as laid down in the Constitution.

92. Solution: b)

When a host is exposed to antigens, which may be in the form of living or dead microbes or
other proteins, antibodies are produced in the host body. This type of immunity is called
active immunity. Active immunity is slow and takes time to give its full effective response.
Injecting the microbes deliberately during immunisation or infectious organisms gaining
access into body during natural infection induce active immunity. When ready-made
antibodies are directly given to protect the body against foreign agents, it is called passive
immunity.

93. Solution: d)

US model is based on very strong states. USSR was based on an authoritarian centre.
Canada is a federal country with a Union bias same as India.

http://insightsonindia.com INSIGHTS Page 30


Insights Mock tests – 2015: test 9 Solutions

94. Solution: a)

95. Solution: a)

The very existence of a State including its territorial integrity is in the hands of Parliament.

The Constitution has certain very powerful emergency provisions, which can turn our
federal polity into a highly centralised system once emergency is declared.

Even during normal circumstances, the central government has very effective financial
powers and responsibilities.

Governor has certain powers to recommend dismissal of the State government and the
dissolution of the Assembly.

Planning is under the Concurrent List – you can find it out because each State has a separate
Planning Department.

96. Solution: c)

Bacteria like Streptococcus pneumoniae and Haemophilus influenzae are responsible for the
disease pneumonia in humans which infects the alveoli (air filled sacs) of the lungs. As a
result of the infection, the alveoli get filled with fluid leading to severe problems in

http://insightsonindia.com INSIGHTS Page 31


Insights Mock tests – 2015: test 9 Solutions

respiration. The symptoms of pneumonia include fever, chills, cough and headache. In
severe cases, the lips and finger nails may turn gray to bluish in colour. A healthy person
acquires the infection by inhaling the droplets/aerosols released by an infected person or
even by sharing glasses and utensils with an infected person.

For Widal test, refer: http://en.wikipedia.org/wiki/Widal_test

97. Solution: c)

There are 5 conditions under which it can be done:

a) To give effect to an international treaty to which India is a signatory

b) Under President‘s rule

c) Under national emergency

d) When two or more states request Parliament to do so (applies to only those states)

e) When Rajya Sabha passes a resolution calling for Parliament to legislate in the state list.

98. Solution: a)

Cancer detection is based on biopsy and histopathological studies of the tissue and blood
and bone marrow tests for increased cell counts in the case of leukemias. In biopsy, a piece
of the suspected tissue cut into thin sections is stained and examined under microscope
(histopathological studies) by a pathologist. Techniques like radiography (use of X-rays), CT
(computed tomography) and MRI (magnetic resonance imaging) are very useful to detect
cancers of the internal organs. Computed tomography uses X-rays to generate a three-
dimensional image of the internals of an object. MRI uses strong magnetic fields and non-
ionising radiations to accurately detect pathological and physiological changes in the living
tissue.

99. Solution: d)

The style of temple architecture that became popular in northern India is known as nagara.
In North India it is common for an entire temple to be built on a stone platform with steps
leading up to it. Further, unlike in South India it does not usually have elaborate boundary
walls or gateways. While the earliest temples had just one tower, or shikhara, later temples
had several. The garbhagriha is always located directly under the tallest tower.

There are many subdivisions of nagara temples depending on the shape of the shikhara.
There are different names for the various parts of the temple in different parts of India;
however, the most common name for the simple shikhara which is square at the base and
whose walls curve or slope inward to a point on top is called the 'latina' or the rekha-
prasada type of shikara.

The second major type of architectural form in the nagara order is the phamsana. Phamsana
buildings tend to be broader and shorter than latina ones. Their roofs are composed of

http://insightsonindia.com INSIGHTS Page 32


Insights Mock tests – 2015: test 9 Solutions

several slabs that gently rise to a single point over the centre of the building, unlike the
latina ones which look like sharply rising tall towers. Phamsana roofs do not curve inward,
instead they slope upwards on a straight incline. In many North Indian temples you will
notice that the phamsana design is used for the mandapas while the main garbhagriha is
housed in a latina building. Later on, the latina buildings grew complex, and instead of
appearing like a single tall tower, the temple began to support many smaller towers, which
were clustered together like rising mountain-peaks with the tallest one being in the centre,
and this was the one which was always above the garbhagriha.

The third main sub-type of the nagara building is what is generally called the valabhi type.
These are rectangular buildings with a roof that rises into a vaulted chamber. The edge of
this vaulted chamber is rounded, like the bamboo or wooden wagons that would have been
drawn by bullocks in ancient times. They are usually called ‗wagonvaulted buildings‘. As
mentioned above, the form of the temple is influenced by ancient building forms that were
already in existence before the fifth century CE. The valabhi type of building was one of
them. For instance, if you study the ground-plan of many of the Buddhist rock-cut chaitya
caves, you will notice that they are shaped as long halls which end in a curved back. From
the inside, the roof of this portion also looks like a wagon-vaulted roof.

100. Solution: a)

The density of a population in a given habitat during a given period, fluctuates due to
changes in four basic processes, two of which (natality and immigration) contribute an
increase in population density and two (mortality and emigration) to a decrease.

(i) Natality refers to the number of births during a given period in the population that are
added to the initial density.

(ii) Mortality is the number of deaths in the population during a given period.

(iii) Immigration is the number of individuals of the same species that have come into the
habitat from elsewhere during the time period under consideration.

(iv) Emigration is the number of individuals of the population who left the habitat and gone
elsewhere during the time period under consideration.

http://insightsonindia.com INSIGHTS Page 33


Insights Mock tests – 2015: test 9 Solutions

http://insightsonindia.com INSIGHTS Page 34


INSIGHTS ON INDIA MOCK PRELIMINARY EXAM - 2015

INSIGHTS ON INDIA MOCK TEST - 9


GENERAL STUDIES
PAPER-I
Time Allowed: 2 Hours Maximum Marks: 200

INSTRUCTIONS
1. IMMEDITELY AFTER THE COMMENCEMENT OF THE EXAMINATION, YOU SHOULD
CHECK THAT THIS TEST BOOKLET DOES NOT HAVE ANY UNPRINTED OR TORN OR MISSING
PAGES OR ITEMS, ETC. IF SO, GET IT REPLACED BY A COMPLETE TEST BOOKLET.
2. You have to enter your Roll Number on the Test
Booklet in the Box provided alongside. DO NOT
Write anything else on the Test Booklet.
4. This Test Booklet contains 100 items (questions). Each item is printed only in English. Each item
comprises four responses (answers). You will select the response which you want to mark on the
Answer Sheet. In case you feel that there is more than one correct response, mark the response which
you consider the best. In any case, choose ONLY ONE response for each item.
5. You have to mark all your responses ONLY on the separate Answer Sheet provided. See directions in
the Answer Sheet.
6. All items carry equal marks.
7. Before you proceed to mark in the Answer Sheet the response to various items in the Test Booklet, you
have to fill in some particulars in the Answer Sheet as per instructions sent to you with your
Admission Certificate.
8. After you have completed filling in all your responses on the Answer Sheet and the examination has
concluded, you should hand over to the Invigilator only the Answer Sheet. You are permitted to take
away with you the Test Booklet.
9. Sheets for rough work are appended in the Test Booklet at the end.
10. Penalty for wrong answers :
THERE WILL BE PENALTY FOR WRONG ANSWERS MARKED BY A CANDIDATE IN THE
OBJECTIVE TYPE QUESTION PAPERS.
(i) There are four alternatives for the answer to every question. For each question for which a
wrong answer has been given by the candidate, one-third of the marks assigned to that
question will be deducted as penalty.
(ii) If a candidate gives more than one answer, it will be treated as a wrong answer even if one of
the given answers happens to be correct and there will be same penalty as above to that
question.
(iii) If a question is left blank, i.e., no answer is given by the candidate, there will be no penalty
for that question.

http://insightsonindia.com

INSIGHTS ON INDIA MOCK TEST SERIES FOR CIVIL SERVICES PRELIMINARY EXAM 2015

http://insightsonindia.com INSIGHTS Page 1


1. Which of the following factors may have 4. A climax community is one where the
helped the growth of population during community is on the verge of
the 19th and 20th century? extinction
1. Sanitation technology
2. Development and spread of Which of the above statements is/are correct?
agriculture a) 1,2 and 4 Only
3. Industrial revolution b) 2,3 and 4 Only
c) 1,2 and 4 only
Choose the correct answer using the codes below:
d) 1,2 and 3 Only
a) 1 and 2
b) 2 and 3 4. Consider the following statements with
c) 1 and 3 respect to the demography of a country.
d) All of the above
Assertion (A): Developed countries are taking
more time to double their population as
2. With reference to cave paintings in India,
compared to developing countries.
consider the following statements
1. The Lakhudiyar cave paintings near Reason (R): There is negative correlation between
Barechhina in Jammu and Kashmir economic development and population
belong to Palaeolithic age growth.
2. Bhimbetka in Madhya Pradesh, where
rich cave paintings have been found, is In the context of the statements above, which of
located in the Vindhya hills these is true?

Which of the above statements is/are correct? a) A and R both are true, and R is the
correct explanation for A.
a) 1 Only b) A and R both are true, and R is the
b) 2 Only NOT the correct explanation for A.
c) Both c) A is correct, R is incorrect.
d) None d) A and R both are incorrect

3. Consider the following statements 5. Consider the following age pyramid and
1. The species that invade a bare area are some conclusions derived from it.
called pioneer species
2. The gradual and fairly predictable
change in the species composition of a
given area is called ecological
succession
3. If succession takes place in areas that
somehow, lost all the living organisms
that existed there, then it is called as
secondary succession

http://insightsonindia.com INSIGHTS Page 2


1. It is the age pyramid of a developing
country. 8. Which of the following is NOT a
2. Population is constant. necessary pillar of human development?
3. A greater part of the population is 1. Equity
between the ages of 15-59 than the 2. Empowerment
other age groups. 3. Sustainability

Which of the following conclusions MAY be Choose the correct answer using the codes below:
correct?
a) 1 and 2
a) 1 and 2 b) 2 and 3
b) 2 and 3 c) 1 and 3
c) 1 and 3 d) All are necessary pillars
d) All of the above
9. With reference to the Lion Capital found
6. Which of the following Indus Valley at Sarnath, consider the following
Civilisation sites is NOT located in statements
Gujarat? 1. It was built in commemoration of the
1. Dholavira historical event of the first sermon or
2. Ropar the Dhammachakrapravartana by the
3. Kalibangan Buddha at Sarnath
4. Rakhigarhi 2. This was built by Ashoka
5. Balathal 3. The crowning element, Dharamchakra,
6. Lothal a large wheel above the lions, was also
Choose the correct answer using the codes below a part of this pillar
(a) 1,2,3 and 4
Which of the above statements is/are correct?
(b) 2,3,4 and 6
(c) 2,3,4 and 5 a) 2 and 3 Only
(d) 2,3 and 4 b) 1 and 2 Only
c) 1 and 3 Only
7. Which of the following characteristics d) All
differentiate between an urban and a rural
area? 10. Which of the following aspects of a
1. Castesim is greater in rural areas. citizen‘s life are included in Bhutan‘s
2. Greater part of the population is Gross National Happiness (GNH) index?
engaged in non-primary activities in 1. Spiritual
an urban area. 2. Cultural
3. Density of population is higher in 3. Economic
rural areas. 4. Good governance
Choose the correct answer using the codes below:
Choose the correct answer using the codes below:
a) 3 and 4 only
a) 1 and 2 b) 2, 3 and 4 only
b) 2 and 3 c) 1 and 2 only
c) 1 and 3 d) All of the above
d) 2 only

http://insightsonindia.com INSIGHTS Page 3


11. Consider the following statements about
the ‗Basic Structure‘ of the Constitution of 14. Which of the following are the
India. characteristic of mixed farming?
1. It is mentioned in Part III of the 1. Fodder crops are an important
Constitution. component.
2. The Supreme Court has interpreted 2. Crop rotation
the Basic Structure in its various 3. Inter-cropping
judgements giving also the exhaustive 4. Extensive use of chemical fertilizers
constituents of the ‗basic structure‘.
3. A law or constitutional amendment Choose the correct answer using the codes below:
can be termed void if it violates the a) 3 and 4 only
‗Basic Structure‘. b) 2, 3 and 4 only
c) 1 and 2 only
Choose the correct answer using the codes below:
d) All of the above
a) 1 and 2
b) 2 and 3 15. Truck farming is common in regions
c) 1 and 3 where farmers specialize in growing
d) 3 only a) Cotton
b) Vegetables
12. With reference to carrying capacity of the c) Flowers
earth, consider the following statements d) Fruits
1. Population size decreases above
carrying capacity of the earth 16. Consider the following statements
2. Ecological Footprint accounting helps 1. The best example Gandhara style is the
in estimating human demand stupa sculptures found at Sanghol in
compared to ecosystem's carrying the Punjab
capacity 2. In the Mathura school of sculpture,
Which of the above statements is/are correct? only Buddhist sculptures are carved
a) 1 Only
Which of the above statements is/are correct?
b) 2 Only
c) Both a) 1 Only
d) None b) 2 Only
c) Both
13. Which of the following factors are d) None
common to countries with a lower HDI
value? 17. How are cooperative and collective
1. Political instability farming different?
2. Social unrest 1. In collective farming, farmers own the
3. Common religion pieces of land collectively, not
Choose the correct answer using the codes below: individually.
a) 1 and 2 2. In collective farming, there is no
b) 2 and 3 hierarchy – all farmers have equal
c) 1 and 3 power – unlike cooperative farming.
d) All of the above

http://insightsonindia.com INSIGHTS Page 4


3. A cooperative can be state enforced 21. Which of the following perform Quinary
unlike collective farming. activities?
Choose the correct answer using the codes below: 1. Secretary of Union Health
a) 1 and 2 Department, Government of India.
b) 2 and 3 2. CEO of an India based MNC
c) 1 and 3 3. Research scientists
d) All of the above Choose the correct answer using the codes below:
a) 1 and 2
18. Industries based on cheap and weight b) 2 and 3
losing material are located close to the c) 1 and 3
sources of raw material because d) All of the above
a) Further transportation cost will be less
b) Processing cost will be less 22. Salmonella typhi which causes typhoid
c) Raw material cost will be less fever in human beings is a
d) Labour cost will be less a) Bacteria
b) Protozoa
19. Consider the following statements about c) Virus
foot lose industries. d) Fungus

Assertion (A): They can be located in a


23. Consider the following statements
wide variety of places. 1. Commensalism is the interaction in
Reason (R): Because of their non- which one species benefits and the
dependence on raw materials. other is neither harmed nor benefited
2. A grazing cattle is a classic example of
In the context of the statements above, which of commensalism
these is true? Which of the above statements is/are correct?
a) 1 Only
a) A and R both are true, and R is the
b) 2 Only
correct explanation for A.
c) Both
b) A and R both are true, and R is the
d) None
NOT the correct explanation for A.
c) A is correct, R is incorrect.
24. When outsourcing involves transferring
d) A and R both are incorrect
work to overseas locations, it is described
by the term off – shoring. Why do
20. Which of the following characteristics do
companies off-shore?
NOT belong to the Agri-businesses?
a) Non-availability of skilled manpower
1. Highly mechanised
in the country that off-shores
2. Large in Size
b) Availability of cheap skilled labour in
3. Reliance on chemicals
the country that is being off-shored to
Choose the correct answer using the codes below:
c) Lack of good business climate in the
a) 1 and 2
country that off-shores
b) 2 and 3
d) To avoid issues of transfer pricing to
c) 1 and 3
evade taxes
d) All of the above

http://insightsonindia.com INSIGHTS Page 5


25. Railways in Russia are preferred much a) 1 and 2
more than Highways. What can be the b) 2 and 3
possible reason? c) 1 and 3
a) Huge geographical extent of Russia d) All of the above
b) The extremely cold climate of Russia
c) It is difficult to construct roads in the 29. The wave of globalization has resulted in
Russian terrain an unprecedented growth of international
d) None of the above is an appropriate trade. International trade is based on
reason which of the following principles?
1. Comparative advantage
26. Before the construction of the Suez Canal 2. Complementarity and transferability
which route connected Liverpool (in UK) of goods and services
and Colombo (in Sri Lanka) by sea? 3. Sovereignty
a) The Mediterranean–Indian Ocean Choose the correct answer using the codes below:
Route a) 1 and 2
b) The Cape of Good Hope Sea Route b) 2 and 3
c) The Northern Atlantic Sea Route c) 1 and 3
d) The South Pacific Sea Route d) All of the above

27. Which of the following routes fall under 30. Arrange India‘s total exports (in money
the National Inland Waterways of India? terms) with the following countries in
1. Bhagirathi – Hooghly river route decreasing order.
2. Sadiya–Dhubri stretch of 1. USA
the Brahmaputra River 2. China
3. Lakhipur-Bhanga stretch of Barak 3. UAE
River 4. Brazil
Choose the correct order from the codes below:
Choose the correct answer using the codes below: a) 2134
a) 1 and 2 b) 1234
b) 2 and 3 c) 1342
c) 1 and 3 d) 1324
d) All of the above
31. In a country, a greater aged population is
28. Consider the following statements about problematic for which of the following
the Silk Road. reasons?
1. Trade on this route passed between 1. Low demographic dividend
Europe and China, through India. 2. High social security expenditure
2. There is a recent proposal to establish 3. High death rates in population
special economic zones and industrial
Choose the correct answer using the codes below:
parks in the areas connected by Silk
Road. a) 1 and 2
3. Only Silk was traded on this route. b) 2 and 3
c) 1 and 3
Choose the correct answer using the codes below:
d) All of the above

http://insightsonindia.com INSIGHTS Page 6


1. Countries cannot normally
32. With reference to temple architecture in discriminate between their trading
India, consider the following statements partners.
1. According to some scholars the Vesar 2. A special privilege like lower customs,
style of temples as an independent if granted to one country has to be
style created through the selective extended to other countries too.
mixing of the Nagara and Dravida 3. Countries can not set up a free trading
orders zone with regional countries.
2. The vahan refers to the entrance to the
temple which may be a portico or Choose the correct answer from the codes below:
colonnaded hall that incorporates a) 1 and 2
space for a large number of b) 2 and 3
worshippers c) 1 and 3
d) All of the above
Which of the above statements is/are correct?

a) 1 Only 36. The practice of dumping concerning


b) 2 Only international trade refers to
c) Both a) discrimination meted out by the host
d) None country to the other investing (and
33. The following commodities are the most trading) country
exported from India. b) Very high duties imposed by the host
1. Petroleum Products country on the products of other
2. Pharmaceutical Products countries
3. Cotton, yarn and fabrics c) Foreign countries selling goods at
4. Electronics items unreasonably lower prices in
international markets.
Arrange them in decreasing order from the codes
d) None of the above
below:

a) 2134 37. WTO does NOT cover which of the


b) 1234 following areas of international trade?
c) 1243 1. Intellectual Property Rights
d) 2143 2. Trade in ‗Invisibles‘
3. Banking
34. A narrow base and broad top in an age
Choose the correct answer from the codes below:
pyramid signifies
a) Population ageing a) 1 and 2
b) Population is young b) 2 and 3
c) Death rates are very low. c) 3 only
d) Both (a) and (c) d) All of the above are covered by WTO

35. Which of the following are covered under


the Most-favoured nation (MFN)
provision of WTO?

http://insightsonindia.com INSIGHTS Page 7


38. Which of the following components are 1. They are used to treat depression,
included in WHO‘s definition of a anxiety and stress.
‗Healthy City‘? 2. They can also be used to treat major
1. Meets the ‗Basic Needs‘ of ‗All‘ its states of mental disturbance in
inhabitants. schizophrenics and other psychotic
2. Involves the ‗Community‘ in local patients.
government. 3. They are highly useful in alleviating
3. Upgrading ‗Energy‘ use and the delusions, hallucinations, and
alternative ‗Transport‘ systems. disordered thinking.

Choose the correct answer from the codes below: Choose the correct answer from the codes below:

a) 1 and 2 a) 1 and 2
b) 2 and 3 b) 2 and 3
c) None of the above c) 1 and 3
d) All of the above d) All of the above

39. Consider the following statements about 42. Consider the following statements about
Antacids. Public Interest Litigations (PILs).
1. They are used to treat acidity in the 1. Only the affected
liver and pancreas. individual/group/institution can file a
2. Excessive administration of Antacids PIL in the court.
can trigger the production of more 2. PILs are mentioned in Article 144 of
acid in the concerned organ(s). the constitution of India to ensure
3. Antacids can also be used to treat ant social justice to the marginalised.
bites. 3. Judiciary can consider a case on its
own based on a newspaper report or
Choose the correct answer from the codes below:
postal complaint received by the court.
a) 1 and 2
Choose the correct answer using the codes below:
b) 2 and 3
c) 1 and 3 a) 1 and 2
d) All of the above b) 2 and 3
c) 1 and 3
40. You are suffering with nasal congestion d) 3 only
associated with common cold and allegy
due to pollen grains. Which of the 43. Chikungunya, an infection caused by the
following is most likely to treat your Chikungunya virus, is transmitted to
congestion? humans by
a) Anti-histamines a) Drinking infected water
b) Analgesics b) Bite of specific species of infected
c) Peptides mosquitos
d) None of the above c) Coming in contact with secretions of
infected person
41. Consider the following statements about d) None of the above
Tranquilizers.

http://insightsonindia.com INSIGHTS Page 8


44. Analgesics are used to reduce or abolish 2. Judges cannot be removed without an
pain. But their use in human beings can inquiry by a bench of the Supreme
lead to the impairment of Court.
1. Mental Confusion 3. Absolute majority is required in the
2. Incoordination in the nervous system Parliament to remove judges.
3. Impairment of Consciousness
Choose the correct answer using the codes below:
Choose the correct answer from the codes below:
a) 1 and 2
a) 1 and 2 b) 2 and 3
b) 2 and 3 c) 1 and 3
c) 1 and 3 d) All of the above
d) None of the above
48. Consider the following statements:
45. Excessive and inappropriate use of anti- 1. Disinfectants can be applied to both
biotics leads to anti-microbial resistance. human body and inanimate objects.
Which of the following processes may 2. By varying the concentration, same
lead to anti-microbial resistance after the substance can act as a disinfectant or
application of anti-biotics? antiseptic.
a) Natural Selection
Which of these is/are true?
b) Budding
c) Selective assimilation by the body a) 1 only
d) Selective assimilation by the bacteria b) 2 only
c) Both 1 and 2
46. Consider the following statements d) None of the above
1. The exploitative appropriation of
indigenous forms of knowledge by 49. Antifertility drugs are used for birth
commercial actors is known as control for family planning. How do anti-
biopiracy fertility drugs work?
2. The Nagoya Protocol to the a) By suppressing sexual hormone
Convention on Biological Diversity formation in the body
regulates biopiracy related issues b) By suppressing ovulation in the body
c) By causing chemical sterilization
Which of the above statements is/are correct?
d) All of the above
a) 1 Only
b) 2 Only 50. Consider the following statements:
c) Both 1. The Size of the territory of a country
d) None plays an important role in determining
the level of human development.
47. Consider the following statements about 2. The higher the per capita income the
the removal of judges of Supreme Court higher will be the HDI of a nation.
and High Courts. 3. In India, among states, higher per
1. Motion to impeach a judge is admitted capita income states always perform
by the respective chairman/speaker of better at HDI than other states with a
the houses of Parliament. lower per capita income.

http://insightsonindia.com INSIGHTS Page 9


Choose the correct answer using the codes below: b) A and R both are true, and R is the
NOT the correct explanation for A.
a) 1 and 2
c) A is correct, R is incorrect.
b) 2 and 3
d) Both A and R are incorrect
c) 3 only
d) None of the above
54. Food preservatives prevent spoilage of
food due to microbial growth. Which of
51. Consider the following additives to food.
the following can NOT be used as a food
1. Fat emulsifiers and stabilising agents
preservative?
2. Antioxidants
1. Sugar
3. Preservatives 2. Vegetable Oil
Which of the above has/have nutritional value? 3. Sodium Benzoate

a) 1 and 2 Choose the correct answer from the codes below:


b) 2 only
a) 1 and 2
c) 1 and 3
b) 2 and 3
d) None of the above
c) 1 and 3
d) All of the above can be used as
52. In a federation
preservatives
1. No more than two autonomous tiers of
government can exist.
55. Consider the following statements about
2. All tiers derive their authority from
soaps used in daily life:
the higher tier and the constitution.
1. Shaving soaps contain ethanol to
3. Each tier has distinct set of powers and
prevent rapid drying.
responsibilities.
2. Soaps containing glycerol are
Choose the correct answer using the codes below: transparent.
3. Beating tiny air bubbles before
a) 1 and 2 hardening makes soaps float on water.
b) 2 and 3
c) 1 and 3 Choose the correct answer from the codes below:
d) 3 only
a) 1 and 2
b) 2 and 3
53. Consider the following statements:
c) 1 and 3
Assertion (A): No artificial sweetener can d) 3 only
be used for cooking (by heating) purposes.
56. Consider the following statements
Reason (R): Artificial sweeteners turn toxic 1. DNA molecules are positively charged
at higher temperatures. molecules
2. Gel electrophoresis technique is used
In the context of the statements above, which of
in separating the fragmented DNA
these is true?
molecules
a) A and R both are true, and R is the
Which of the above statements is/are correct?
correct explanation for A.

http://insightsonindia.com INSIGHTS Page 10


a) 1 Only a) 2134
b) 2 Only b) 1234
c) Both c) 2143
d) None d) 1243

57. Consider the following statements about 60. Consider the following statements about
the powers of the Supreme Court of India. the North Eastern Region Power System
1. Its decisions are binding on all courts. Improvement Project (NERPSIP) recently
2. It can transfer judges of High Courts. approved by the Government of India.
3. It can move cases from any court in 1. Half of the funding of the project is
India to itself. from the World Bank.
4. It can transfer cases from one High 2. The main aim of the project is to
Court to another. increase the total power production in
the NE region.
Choose the correct answer using the codes below: 3. It extends to Sikkim and Arunachal
a) 1 and 2 only Pradesh also.
b) 2 and 3 only
Choose the correct answer from the codes below:
c) 1, 2 and 3 only
d) All of the above a) 1 and 2
b) 2 and 3
58. Sometimes dyes do not get easily c) 1 and 3
absorbed on the clothe piece. This can be d) 1 only
due to
a) Clothe has been put in sunlight for 61. Consider the following statements
long. 1. The caves of Ellora and Aurangabad
b) Clothe has been washed with hard show the ongoing differences between
water. the two religions—Buddhism and
c) Clothe has been washed with alcohol Brahmanical
and then dried and dyed. 2. Triple storey cave carving is found in
d) Clothe and Dye may not be at the Ajanta whereas it is absent in Ellora
same temperature caves

Which of the above statements is/are correct?


59. Renewable energy can also play an
important role in resolving the energy a) 1 Only
crisis in urban areas to a great extent. b) 2 Only
Consider the following sources of c) Both
renewable energy in India. d) None
1. Bio power
2. Small Hydro power 62. Which of the following is NOT captured
3. Solar power in the Human Development index (HDI)?
4. Wind Power 1. Sanitation coverage
2. Assets of a household
Arrange the following in increasing order of their
3. Women labour participation rate
present production in India.

http://insightsonindia.com INSIGHTS Page 11


Choose the correct answer using the codes below: d) All of the above

a) 1 and 2
65. The Cabinet has recently given its
b) 2 and 3
approval to ratify Beijing Protocol and
c) 1 and 3
introduce a bill in the light of the protocol.
d) None is captured
The Beijing protocol concerns
a) Hijacking of planes
63. As some economists point out, India is on
b) International Aviation operational
a threshold of an Investment Recovery
safety standards
Cycle. Which of the recent decisions of the
c) Safety standards on International
GoI would help in it?
airports
1. Increasing FDI caps in defense and
d) International cooperation for better
insurance sectors
operation of international flights
2. Recent ordinance on coal block
auctions
66. Consider the following statements
3. Make in India campaign
1. Antibodies are produced by T-
Choose the correct answer from the codes below: lymphocytes as part of primary
response to pathogens
a) 1 and 2 2. Acquired immunity is pathogen
b) 2 and 3 specific and is characterised by
c) 1 and 3 memory
d) All of the above
Which of the above statements is/are correct?
64. The traditional utensil making among
a) 1 Only
the Thatheras of Jandiala Guru, Punjab is
b) 2 Only
being inscribed on the UNESCO
c) Both
Representative List of
d) None
the Intangible Cultural Heritage of
Humanity, 2014. Consider the following
67. Consider the following statements
statements about it.
1. Restriction endonucleases which cut
1. The utensil is an alloy of copper, zinc
DNA at specific site are basically
and tin.
enzymes
2. The tradition of using the metals is
2. Restriction endonucleases are isolated
recommended by the ancient Indian
from bacteria
school of medicine, Ayurveda.
3. Skills of the Thatheras have been Which of the above statements is/are correct?
transmitted in written instructions
a) 1 Only
from one generation to other
b) 2 Only
generations in Punjab.
c) Both
Choose the correct answer from the codes below: d) None

a) 1 and 2
b) 2 and 3
c) 1 and 3

http://insightsonindia.com INSIGHTS Page 12


68. The Constitution (119th Amendment) Bill, 1. Health food
2013 aims to ratify the LBA between India 2. Textile industry
and Bangladesh under the Indira-Mujib 3. Biodegradable plastics
pact of 1974 to exchange areas and people
on either side of the border. Which of the Choose the correct answer using the codes below:
following states will be involved in the a) All of the above
land transfer? b) 1 and 2
1. Assam c) 1 and 3
2. Nagaland d) 2 and 3
3. Meghalaya
4. Tripura 71. The RBI recently issued the final
guidelines for the Bharat Bill Payment
Choose the correct answer using the codes below:
System (BBPS). Consider the following
a) All of the above statements about BBPS.
b) 1 only 1. No agents are involved in BBPS as it is
c) 1, 3 and 4 a direct customer service.
d) 2 and 4 only 2. Multiple bills can be paid at single
point of transaction.
69. The Prime Minister recently inaugurated 3. Banking operations have been
the Hornbill festival in one of the North- excluded from BBPS.
eastern states. Consider the following
Choose the correct answer using the codes below:
statements about it.
1. It is a tourism promotional a) 1 and 2
extravaganza of Nagaland. b) 2 and 3
2. It is also supported by the Centre as c) 1 and 3
one of the biggest d) 2 only
indigenous festivals of the country.
3. The festival is organized by the State 72. Consider the following statements about
Tourism and Art & Culture Mount Aso.
Departments. 1. It was an active volcano few decades
back, but is an inactive volcano now.
Choose the correct answer using the codes below:
2. It is located in Japan.
a) All of the above 3. The explosions in Mount Aso are of
b) 1 and 2 very low intensity (if any).
c) 1 and 3
Choose the correct answer using the codes below:
d) 2 and 3
a) 1 and 2
70. India‘s first annual hemp conference was b) 2 and 3
held in New Delhi at Indian Habitat c) 1 and 3
Centre (IHC). The aim of the conference d) 2 only
was to provide an opportunity to
international agencies get idea of India‘s
hemp agencies. In which of the following
can hemp be used?

http://insightsonindia.com INSIGHTS Page 13


73. The Lok Sabha has recently passed distribution of powers laid down by
the Constitution (Scheduled Castes) the Constitution.
Orders (Amendment) Bill, 2014. Which of 3. The review power extends to the laws
the following communities have been passed by State legislatures also.
added to the SC list?
1. Pulluvan and Thachar community Choose the correct answer using the codes below:
from Kerala a) 1 and 2
2. Dhobi community from Tripura b) 2 and 3
3. Majhi community in Sikkim c) 1 and 3
d) All of the above
Choose the correct answer using the codes below:

a) 1 and 2 76. Consider the following statements with


b) 2 and 3 reference to plasmids which are used in
c) 1 and 3 gene cloning
d) All of the above 1. It is an independently replicating
extra-chromosomal DNA molecule
74. The World Trade Organization (WTO) 2. They are most commonly found in
approved the first world-wide trade deal- bacteria as small, circular, double-
Trade Facilitation agreement (TFA). stranded DNA molecules
Which of the following are included in
Which of the above statements is/are correct?
TFA?
1. Lowering import tariffs and a) 1 Only
agricultural subsidies for developing b) 2 Only
countries c) Both
2. Developed countries would abolish d) None
hard import quotas on agricultural
products from the developing world. 77. Exotic species can affect the local
3. Reduction in red tape ecosystem in which of the following
at international borders ways?
1. Compete with native species for
Choose the correct answer using the codes below:
resources
a) 1 and 2 2. Alter the ecosystem to the
b) 2 and 3 disadvantage of other species
c) 1 and 3 3. Cause loss of bio-diversity
d) All of the above
Choose the correct answer using the codes
below:
75. Consider the following statements about
Judicial review. a) 1 and 2
1. The term ‗judicial review‘ is b) 2 and 3
mentioned nowhere in the c) 1 and 3
Constitution. d) All of the above
2. Supreme Court can use the review
powers if a law is inconsistent with the

http://insightsonindia.com INSIGHTS Page 14


78. Which of the following are the major Choose the correct answer using the codes
points of contentions at the ongoing below:
Climate change conference at Lima, Paris?
a) 1 and 2
1. Long-term financing for tackling
b) 2 and 3
climate change
c) 1 and 3
2. Principle of Common But
d) 2 only
differentiated Responsibilities
3. Historic emissions issues of developed
81. Few countries have recently banned ‗Dirty
countries
Coal‘. Consider the following statements
Choose the correct answer using the codes about ‗Dirty Coal‘.
below: 1. It has higher ash content than normal
coking coal.
a) 1 and 2
2. It has lower sulphur content than
b) 2 and 3
normal coking coal.
c) 1 and 3
3. Dirty coal is one of the reasons behind
d) 1 only
formation of smog in urban areas.

79. Watershed conservation is one of the Choose the correct answer using the codes below:
major interventions to fight the shortage
a) 1 and 2
of water in many regions of India. What
b) 2 and 3
else is included in Watershed
c) 1 and 3
Conservation?
d) All of the above
1. Forest protection
2. Agricultural best management
82. The Union Ministry of New and
practices
Renewable Energy (MNRE) recently has
3. Soil Conservation
announced a scheme to develop 25 solar
Choose the correct answer using the codes parks across the country. Consider the
below: following statements.
1. There are no solar parks in India till
a) 1 and 2
date.
b) 2 and 3
2. In the scheme, no solar parks are
c) 1 and 3
proposed in the North-Eastern region
d) All of the above
of India.
3. If the scheme is implemented
80. Recently ‗Social Justice Bench‘ has been set
successfully, the target under the
up by the Judiciary to hear cases related to
Jawaharlal Nehru National Solar
social issues. Consider the following about
Mission will be accomplished before
it.
target year.
1. It has been setup in the Supreme Court
as well as all the High Courts of India. Choose the correct answer using the codes below:
2. The Bench will dispose off PILs.
a) 1 and 2
3. It will be headed by the Chief Justices
b) 3 only
of the respective court(s).
c) 1 and 3

http://insightsonindia.com INSIGHTS Page 15


d) 2 and 3 2. Installing giant light reflecting mirrors
on earth
83. Tigers are at the apex of the foodweb in a 3. Cloud Seeding and cloud whitening
forest ecosystem. Consider the following
statements about the status of Tigers in Choose the correct answer using the codes below:
India. a) 1 and 2
1. As per the latest reports, highest b) 2 and 3
numbers of tigers have died recently in c) 1 and 3
the last four years than ever between d) All of the above
any two censuses.
2. There is no policy for protection of 86. Which of the following would be affected
tigers outside reserves from the if India does not have an Independent
Central government. Judiciary?
1. Rule of Law and Supremacy of Law
Which of these is/are true?
2. Enforcement of Fundamental Rights
a) 1 only 3. Legitimacy of Indian democracy
b) 2 only
Choose the correct answer using the codes below:
c) Both 1 and 2
d) None a) 1 and 2
b) 2 and 3
84. Consider the following statements c) 1 and 3
1. The rock-cut cave carved at Barabar d) All of the above
hills near Gaya in Bihar is known as
the Lomus Rishi cave was patronised 87. With reference to BOD (biochemical
by Ashoka for the Ajivika sect oxygen demand), consider the following
2. Most of the Buddhist stupas are statements
constructed over the relics of the 1. The lesser the BOD of waste water,
Buddha more is its polluting potential
2. The sewage water is treated till the
Which of the above statements is/are
BOD is increased to make it useful for
INCORRECT?
consumption
a) 1 Only
Which of the above statements is/are
b) 2 Only
INCORRECT?
c) Both
d) None a) 1 Only
b) 2 Only
85. Scientists have warned that c) Both
geoengineering, an emerging discipline d) None
that seeks to tackle problems related to
climate change, could have ―catastrophic
consequences‖ for the planet‘s inhabitants.
Which of the following come under geo-
engineering?
1. Ocean iron fertilization

http://insightsonindia.com INSIGHTS Page 16


88. Which of the following is/are a part of an 2. Square or rectangular copper tablets,
Independent Judicial system? with an animal or a human figure on
1. Judiciary is not accountable to the one side and an inscription on the
legislature. other, or an inscription on both sides
2. Judicial appointments are not affected have also been found
solely by political considerations.
Which of the above statements is/are correct?
3. The Executive is bound to follow and
implement the decisions of the a) 1 Only
Judiciary even if it disagrees with it. b) 2 Only
c) Both
Choose the correct answer using the codes below:
d) None
a) 1 and 2
b) 2 and 3 91. Which of the following would come under
c) 1 and 3 the Original Jurisdiction of the Supreme
d) All of the above Court of India?
1. Advising the President on matters of
89. Consider the following statements about public importance and law.
the Judiciary in India. 2. Granting special leave to an appeal
1. The judiciary is not financially from any judgement or matter passed
dependent on either the executive or by any court in the territory of India.
legislature. 3. Settling disputes between Union and
2. Parliament cannot discuss the conduct States and amongst States.
of the judges except when the
Choose the correct answer using the codes below:
proceeding to remove a judge is being
carried out. a) 1 and 2
3. The judiciary has the power to b) 2 and 3
penalise those who are found guilty of c) 3 only
contempt of court in case of unfair d) 1 and 3
criticism of its decisions.
92. When ready-made antibodies are directly
Choose the correct answer using the codes below:
given to protect the body against foreign
a) 1 and 2 agents, it is called
b) 2 and 3 a) Active immunity
c) 1 and 3 b) Passive immunity
d) All of the above c) Response immunity
d) None of the above
90. With reference to seals and tablets found
at Indus Valley Civilization sites, consider 93. Indian model of federation is closer to that
the following statements of which of the following nations/nation-
1. The standard Harappan seal was a states?
square plaque 2×2 square inches, 1. USSR
usually made from mud and then 2. Canada
burnt to harden it 3. USA

http://insightsonindia.com INSIGHTS Page 17


Choose the correct answer using the codes below: Which of the above statements is/are correct?

a) 1 and 2 a) 1 Only
b) 2 and 3 b) 2 Only
c) 1 and 3 c) Both
d) 2 only d) None

94. Which of the following fall in the 97. Under which of the following
Concurrent List under the Seventh circumstances can the Parliament legislate
Schedule of the Constitution of India? on the subjects in the State list?
1. Forests 1. Rajya Sabha passing a resolution to
2. Education that effect.
3. Trade and Commerce 2. Financial emergency
4. Ports 3. President‘s Rule.

Choose the correct answer using the codes below: Choose the correct answer using the codes below:

a) 1 and 2 only a) 1 and 2


b) 2 and 3 only b) 2 and 3
c) 1, 2 and 3 only c) 1 and 3
d) All of the above d) All of the above

95. Which of the following provisions of the 98. Consider the following statements
constitution point towards the Central 1. Computed tomography (CT) uses X-
bias in the Indian federation? rays to generate a three-dimensional
1. Emergency provisions image of the internals of an object
2. The Position of Governor 2. MRI uses strong magnetic fields and
3. Constitution provides ‗Planning‘ non-ionising radiations to accurately
under the Union List. detect pathological and physiological
changes in the living tissue
Choose the correct answer using the codes below: 3. CT is useful in detecting cancer in
a) 1 and 2 inner organs whereas MRI is not
b) 2 and 3 useful in detecting cancers
c) 1 and 3
Which of the above statements is/are correct?
d) All of the above
a) 1 and 2 Only
96. Consider the following statements b) 1 and 3 Only
1. In Widal test, bacteria causing typhoid c) 2 and 3 Only
fever are mixed with serum containing d) All
specific antibodies obtained from an
infected individual
2. Bacteria like Streptococcus pneumoniae
and Haemophilus influenza cause lung
diseases in humans

http://insightsonindia.com INSIGHTS Page 18


99. With reference to nagara style of temple
architecture, consider the following 100. Which of the following processes
statements contribute in an increase of population of
1. It is popular in northern India a region?
2. It does not usually have elaborate 1. Natality
boundary walls or gateways 2. Immigration
3. The garbhagriha is always located 3. Emigration
directly under the tallest tower 4. Mortality
4. The valabhi type – the rectangular
buildings with a roof that rises into a Choose the answer using the codes below
vaulted chamber, is a sub-type of a) 1 and 2 Only
nagara style b) 1 and 3 Only
Which of the above statements is/are correct? c) 2 and 3 Only
a) 1,2 and 3 Only d) 2 Only
b) 2,3 and 4 Only
c) 1,3 and 4 Only
d) All

http://insightsonindia.com INSIGHTS Page 19

You might also like